Calculating Outward Flux of Vector Field on Ball Boundary

Click For Summary
The discussion revolves around calculating the outward flux of a two-dimensional vector field through the boundary of a ball using the divergence theorem. The divergence of the vector field was calculated to be 1, leading to an initial flux result of πR². However, an alternative method using the unit normal and parameterization of the circle yielded a different result of πR. The discrepancy arises from a misunderstanding regarding the integral's variable, where the correct differential should be ds, representing arc length, rather than dt. Clarification on this point is essential for obtaining the correct flux calculation.
kidsmoker
Messages
85
Reaction score
0

Homework Statement



Calculate the outward flux of the two dimensional vector field

f:\Re^{2}\rightarrow\Re^{2} , f(x,y)=(x/2 + y\sqrt{x^{2}+y^{2}},y/2 + x\sqrt{x^{2}+y^{2}})

through the boundary of the ball

\Omega = {(x,y)\in\Re^{2} \left| x^{2}+y^{2} \leq R^{2}} \subset\Re^{2}, R>0 .

Homework Equations



Divergence theorem:

\int F.n ds = \int\int divF dA

where the first integral is round the boundary and the second one is over the area (sorry I can't get latex to display the limits of integration).

The Attempt at a Solution



I calculated divF to be 1 which gives the answer to be just pi*R^2 .

I then thought i'd try it the old fashioned way, dotting the unit normal with the field around the edge of the circle. The unit normal is n=(x,y)/R right? So dotting this with the field gives me (x^2+y^2)/2R . Then I changed from x,y to using the angle t around the circle, so x=Rcost, y=Rsint .

This gives f.n= R^2/(2R) = R/2 so the flux is the integral from zero to 2pi of

\int (R/2) dt = (R/2)(2\pi) = \pi R .

I must have gone wrong somewhere cos my answers are different, but I can't spot where :-(

Thanks for any help!
 
Last edited:
Physics news on Phys.org
Your path integral shouldn't be dt, it should be ds where ds is arc length along the path. The length of the curve is 2piR.
 
Question: A clock's minute hand has length 4 and its hour hand has length 3. What is the distance between the tips at the moment when it is increasing most rapidly?(Putnam Exam Question) Answer: Making assumption that both the hands moves at constant angular velocities, the answer is ## \sqrt{7} .## But don't you think this assumption is somewhat doubtful and wrong?

Similar threads

  • · Replies 6 ·
Replies
6
Views
2K
  • · Replies 7 ·
Replies
7
Views
2K
  • · Replies 6 ·
Replies
6
Views
2K
Replies
2
Views
2K
  • · Replies 3 ·
Replies
3
Views
2K
  • · Replies 9 ·
Replies
9
Views
2K
Replies
5
Views
1K
  • · Replies 8 ·
Replies
8
Views
2K
  • · Replies 3 ·
Replies
3
Views
2K
  • · Replies 8 ·
Replies
8
Views
2K